Đến nội dung

Kagome nội dung

Có 161 mục bởi Kagome (Tìm giới hạn từ 27-04-2020)



Sắp theo                Sắp xếp  

#690476 a, cho a,b,c $\in \begin{bmatrix} 0 &;1 \en...

Đã gửi bởi Kagome on 13-08-2017 - 23:18 trong Phương trình - hệ phương trình - bất phương trình

b, cho $\frac{a}{b}< \frac{c}{d}.$

cmr $\frac{a}{b}< \frac{ab+cd}{b^{2}+d^{2}}< \frac{c}{d}$

và $\frac{a}{b}< \frac{a+c}{b+d}< \frac{c}{d}$ nếu $b,d> 0$

Chỉ cm dc mỗi vế thứ 2:

Vì b,d >0 nên b+d>0

$=>a+\frac{ad}{b}<a+c<=>\frac{ad}{b}<c<=>ad<bc$ ( đúng )

tương tự cái bên kia




#690117 Hệ phương trình đẳng cấp phương pháp đồng biến

Đã gửi bởi Kagome on 10-08-2017 - 15:12 trong Phương trình, hệ phương trình và bất phương trình

2y(x2-y2)=3x
x(x2+y2)=10y

viết lại đề nhá: $\left\{\begin{matrix} 2y(x^2-y^2) & = & 3x\\ x(x^2+y^2) & = & 10y \end{matrix}\right.$

Nếu $y=0$ thì $x=0$, nếu $x=0$ thì $y=0$.

Xét $x,y\neq 0$, chia cả 2 vế của pt 1 cho $x^2$, chia cả 2 vế của pt 2 cho $y^2$.

Ta dc:$\left\{\begin{matrix} 2y(1-\frac{y^2}{x^2}) & = & \frac{3}{x}\\ x(\frac{x^2}{y^2}+1) & = & \frac{10}{y} \end{matrix}\right.$

Đặt $\frac{x^2}{y^2}=a$ thì $\frac{y^2}{x^2}=\frac{1}{a}$.

Hpt$<=>\left\{\begin{matrix} xy(1-\frac{1}{a}) & = & \frac{3}{2}\\ xy(1+a) & = & 10 \end{matrix}\right.$

$=>\frac{1-\frac{1}{a}}{a+1}=\frac{3}{20}<=>3a^2-17a+20=0<=>a=4,a=\frac{5}{3}$ 

Khi đó $\frac{x^2}{y^2}=4$ hoặc $\frac{x^2}{y^2}=\frac{5}{3}$.




#686067 Chứng minh tồn tại một số dương trong hai số $2a+b-2\sqrt{cd...

Đã gửi bởi Kagome on 30-06-2017 - 21:22 trong Đại số

Cho $a,b,c,d > 0$. Chứng minh tồn tại một số dương trong hai số $2a+b-2\sqrt{cd}$ và $2c+d-2\sqrt{ab}$

$2a+b-2\sqrt{cd}+2c+d-2\sqrt{ab}=(\sqrt{a}-\sqrt{b})^2+(\sqrt{c}-\sqrt{d})^2+a+c>0=>$đpcm.




#685689 Tìm Min của $P=5(a+b+c)+\frac{3}{abc}$

Đã gửi bởi Kagome on 26-06-2017 - 23:00 trong Bất đẳng thức và cực trị

mình thấy x3$\leq 1\Rightarrow \frac{-2}{x^{3}}\leq -2$ bị ngược dấu mà bạn

Uh, bị ngược dấu rồi, ko để ý.




#685668 Tìm Min của $P=5(a+b+c)+\frac{3}{abc}$

Đã gửi bởi Kagome on 26-06-2017 - 20:01 trong Bất đẳng thức và cực trị

Cho $a^2+b^2+c^2=3$ với a;b;c dương

Tìm Min của $P=5(a+b+c)+\frac{3}{abc}$

$3=a^2+b^2+c^2\geq 3\sqrt[3]{a^2b^2c^2}=>\sqrt[3]{abc}\leq 1$

$P\geq 15\sqrt[3]{abc}+\frac{3}{abc}$

Đặt $x=\sqrt[3]{abc}, 0\leq x\leq 1=>0\leq x^3\leq 1$

$P\geq 15x+\frac{3}{x^3}=3(5x+\frac{1}{x^3}=3(\frac{5}{3}x+ \frac{5}{3}x+\frac{5}{3}x+\frac{5}{3x^3}-\frac{2}{3x^3})\geq 3(\frac{20}{3}-\frac{2}{3})=18$

Dấu = xảy ra $<=> a=b=c=1$

P/S: Giải thế dc ko nhỉ???




#685565 Tìm giá trị nhỏ nhất của biểu thức: A = $\left | 36^{x}-5...

Đã gửi bởi Kagome on 25-06-2017 - 18:28 trong Bất đẳng thức và cực trị

Bạn lập luận như thế là chưa chặt chẽ :

Làm sao có thể khẳng định :

 

+ Nếu $A=1$ ko có giá trị nào của $x,y$ thỏa mãn.

+ Nếu $A=9$ ko có giá trị nào của $x,y$ thỏa mãn.

 

nếu chỉ biết $36^x$ tận cùng là $6$ và $5^y$ tận cùng là $5$ ?

Mình thấy làm kiểu tận cùng này khá ổn rồi. Có cái chỗ khẳng định ko có x,y thỏa mãn thì đúng do mình ẩu thật, có thể chứng minh bằng chia hết giống bạn cũng dc.




#685554 Tìm giá trị nhỏ nhất của biểu thức: A = $\left | 36^{x}-5...

Đã gửi bởi Kagome on 25-06-2017 - 15:06 trong Bất đẳng thức và cực trị

Cho x, y là các số tự nhiên khác 0. Tìm giá trị nhỏ nhất của biểu thức:

A = $\left | 36^{x}-5^{y} \right |$

Vì $36^x$ có tận cùng là 6, $5^y$ có tận cùng là 5 nên $A$ có tận cùng là 1 hoặc 9.

  • Nếu $A=1$ ko có giá trị nào của $x,y$ thỏa mãn.
  • Nếu $A=9$ ko có giá trị nào của $x,y$ thỏa mãn.
  • Nếu $A=11$ thì $x=1,y=2$

Vậy $Min A=11<=>x=1,y=2$.




#685313 Tìm Min của $P=\frac{1}{\sqrt[3]{abc}...

Đã gửi bởi Kagome on 21-06-2017 - 21:43 trong Bất đẳng thức và cực trị

Bài 2:

$\sum \frac{a}{a^3+b^2+c}=\sum \frac{(a)(\frac{1}{a}+1+c)}{(a^3+b^2+c)(\frac{1}{a}+1+c)}\leq \sum \frac{1+a+ac}{(a+b+c)^2}\leq \frac{3+\sum a+ \frac{(\sum a)^2}{3}}{(a+b+c)^2}=1$

Sao bạn nghĩ ra dc bước nhân $\frac{1}{a}+1+c$ hay vậy. Chỗ đó dúng là nghệ thuật luôn á :D




#685310 Chuyên đề phân tích thành nhân tử bằng hệ thức Vi - ét kết hợp với máy tính c...

Đã gửi bởi Kagome on 21-06-2017 - 21:35 trong Phương trình, hệ phương trình và bất phương trình

Dùng máy tính kiểu gì ạ?

Thì em bấm máy ra 2 nghiệm của pt, cộng hai nghiệm ấy lại ra số nguyên, rồi nhân tiếp 2 nghiệm ấy lại, thấy ra số nguyên thì dùng viet như mr cooper ấy. Còn trường hợp ko ra nghiệm nguyên hoặc vô nghiệm thì tìm cách khác.




#685244 Chuyên đề phân tích thành nhân tử bằng hệ thức Vi - ét kết hợp với máy tính c...

Đã gửi bởi Kagome on 21-06-2017 - 12:45 trong Phương trình, hệ phương trình và bất phương trình

Làm sao tính được hệ thức Vi-ét trên ạ? Chỉ em với

Dùng máy tính.




#685145 $\sqrt{a+b}+\sqrt{b+c}+\sqrt{c+a...

Đã gửi bởi Kagome on 20-06-2017 - 14:47 trong Bất đẳng thức và cực trị

Cho các số thực không âm a,b,c. Chứng minh rằng:

P=$\sqrt{a+b}+\sqrt{b+c}+\sqrt{c+a}\leq \frac{3\sqrt{3}}{2}\sqrt{a+b+c-\frac{abc}{ab+bc+ca}}$

Áp dụng bđt $Bunhiacopxki$,ta được: $P^2\leq 6(a+b+c)$.

Cần CM: $6(a+b+c)\leq \frac{27}{4}(a+b+c-\frac{abc}{\sum{ab}})$

$<=>24(a+b+c)\leq 27(a+b+c)-\frac{27abc}{\sum{ab}}$

$<=>3(a+b+c)-\frac{27abc}{\sum{ab}}\geq 0$

Áp dụng bđt $Cauchy$: $\sum{ab}\geq 3\sqrt[3]{a^2b^2c^2}$,$3\sqrt[3]{abc}\leq a+b+c$

$VT\geq 3(a+b+c)-\frac{27abc}{3\sqrt[3]{a^2b^2c^2}}$

      $=3(a+b+c)-9\sqrt[3]{abc}$

      $\geq 3(a+b+c)-3(a+b+c)=0$

Dấu $'='<=> a=b=c$.




#684829 $B=\frac{1}{3}+\frac{5}{7...

Đã gửi bởi Kagome on 17-06-2017 - 20:34 trong Số học

Dùng Casio, ta gõ như sau :)): $\sum_{1}^{504}(\frac{4x-3}{4x-1})$ ra phần nguyên là $500$ :)) kk

đi thi cho làm kiểu vậy ko?




#684736 Cho đa thức $f(x)=x^{2017}+ax^2+bx+c$

Đã gửi bởi Kagome on 16-06-2017 - 21:18 trong Đa thức

19369691_120250815234968_786605732_n.png

M.n giải thích giúp mình phần lí luận thứ 2 của lời giải.

Cái chỗ khung vuông thứ 3 từ trên đếm xuống ấy à?

Nếu là cái chỗ đó thì hình như là do $x_1$ là nghiệm của $f(x)$ nên $f(x_1)=0$ chia hết cho 2017, mà $x^{2017}-x$ chia hết cho 2017 nên $g(x_1)$ chia hết cho 2017




#684358 Chứng minh: $\frac{x}{y+z}+\frac{y...

Đã gửi bởi Kagome on 13-06-2017 - 17:08 trong Bất đẳng thức và cực trị

2) Cho x, y, z là các số dương thỏa mãn: $x^{2}+y^{2}+z^{2}=11$ và $xy+yz+zx=7$. Chứng minh: $\frac{1}{3}\leq x, y, z\leq 3$

Từ gt $=>x+y+z=5$

Ta có $xy+yz+zx=7<=>x(y+z)+yz=7\leq x(y+z)+\frac{(y+z)^2}{4}=x(5-x)+\frac{(5-x)^2}{4}=\frac{-3x^2+10x+25}{4}=>\frac13\leq x\leq 3$.Tương tự...




#683302 Đề thi tuyển sinh lớp 10 chuyên Toán THPT Chuyên Tiền Giang

Đã gửi bởi Kagome on 05-06-2017 - 22:06 trong Tài liệu - Đề thi

Đề năm nay tương đối dễ

Đề dễ mới sợ ấy. Mấy cái đề khó làm 5 điểm còn đậu nổi. Còn mấy cái đề dễ làm 9 điểm ko biết đậu nổi ko.




#683301 Đề thi chuyên Toán vào 10 THPT chuyên Tiền Giang năm học 2017-2018

Đã gửi bởi Kagome on 05-06-2017 - 22:01 trong Tài liệu - Đề thi

Bài PT ta có thể đặt ẩn phụ như sau:

Đặt $\sqrt{x^2+1}=a,\sqrt{x}=b$.

Ta có ngay: $a^2-ab-6b^2=0\Rightarrow a=3b$.

Từ đó ta có thể giải tiếp đơn giản mà ko cần sợ tách ko được.

Làm thế này cũng được nữa:

$PT\Leftrightarrow x^2+1-\sqrt{x(x^2+1)}-6x=0$

Chia cả 2 vế cho x được 

$\frac{x^2+1}{x}-\sqrt{\frac{x^2+1}{x}}-6=0$

Đặt $\sqrt{\frac{x^2+1}{x}}=a$...

P/s: Cái đề này thấy mấy đứa bạn đứa nào cũng giải được hết có mình bỏ một câu. Ko biết đậu nổi ko.




#680252 Topic ôn thi hình học vào cấp 3 chuyên

Đã gửi bởi Kagome on 11-05-2017 - 00:30 trong Hình học

Bài 83: (Sưu tầm)

Cho tam giác $ABC$ nhọn. Các đường cao $BB_{1}; CC_{1}$. Gọi $O$ là trung điểm $BC$. Gọi $M = B_{1}C_{1} \cap BC$; $ P = (BOC_{1}) \cap (COB_{1}) $.Chứng minh rằng $MP; BB_{1}; CC_{1}$ đồng quy tại $1$ điểm.

 

attachicon.gif83.png

  • Chứng minh $A,P,O$ thẳng hàng $\Rightarrow \angle HPO=90^o$

Dễ thấy 4 điểm $A,C_1,H,B_1$ cùng thuộc 1 đường tròn.

Xét tứ giác $AC_1PB_1$ có $\angle AB_1P=\angle POC,\angle AC_1P=\angle POB$ (do 2 tứ giác $C_1POB,B_1POC$ nội tiếp), $\angle POB+\angle POC=180^o \Rightarrow \angle AC_1P+\angle AB_1P=180^o\Rightarrow AC_1PB_1$ nội tiếp.

$\Rightarrow \angle APH=\angle AB_1H=90^o$

Ta có $AC_1.AB=AB_1.AC \Rightarrow B_1C_1BC$ nội tiếp $\Rightarrow \angle APB_1=\angle AC_1B_1=\angle ACB$.

Mà $\angle OCB_1+\angle OPB_1=180^o \Rightarrow \angle APB_1+\angle OPB_1=180^o \Rightarrow A,P,O$ thẳng hàng.

$\Rightarrow \angle HPO=90^o$.                                 (1)

  • Chứng minh $\angle B_1C_1P=\angle PCM\Rightarrow C_1PCM$ nội tiếp.

Ta có $\angle PC_1B_1=\angle PAB_1=\angle PAC$

Ta có $\angle OPC=\angle OB_1C=\angle B_1CO$ ($\angle OB_1C=\angle B_1CO$ do $B_1O=OB=OC$ trung tuyến ứng với cạnh huyền)

Mà $\angle OPC=\angle PAC+\angle ACP, \angle B_1CO=\angle PCO+\angle ACP$ 

$\Rightarrow \angle PAC=\angle PCO$

$\Rightarrow \angle B_1C_1P=\angle PCM \Rightarrow C_1PCM$ nội tiếp

$\Rightarrow \angle MC_1C=\angle MPC \Rightarrow 90^o+\angle MC_1B=\angle MPO+\angle OPC$

Mà $\angle MC_1B=\angle AC_1B_1=\angle B_1CO=\angle OB_1C=\angle OPC$

$\Rightarrow \angle MPO=90^o$                                  (2)

(1)(2)$\Rightarrow M,H,P$ thẳng hàng.

Vậy $MP,BB_1,CC_1$ đồng quy tại 1 điểm.

Chứng minh đồng quy.PNG




#679118 $B=\frac{a}{a^2+2b+3}+\frac{b}...

Đã gửi bởi Kagome on 01-05-2017 - 14:17 trong Bất đẳng thức và cực trị

Cho $a,b,c$ là các số thực dương thỏa: $a^2+b^2+c^2=3$. Tìm GTLN:

$B=\frac{a}{a^2+2b+3}+\frac{b}{b^2+2c+3}+\frac{c}{c^2+2a+3}$.




#676434 giải phương trình $\sqrt{x+3}+\sqrt{3x+1}=...

Đã gửi bởi Kagome on 06-04-2017 - 18:48 trong Đại số

1. $\sqrt{x+3}+\sqrt{3x+1}=\sqrt{2x+1}+\sqrt{2x+3}$

Bình phương hai vế, ta được:

$(x+3)(3x+1)=(2x+1)(2x+3)$

$\Rightarrow -x(x-2)=0$




#676421 Đề thi thử AMSTERDAM Hà Nội vòng 2 2016-2017

Đã gửi bởi Kagome on 06-04-2017 - 16:51 trong Tài liệu - Đề thi

Đề thi thử hôm nay .

17800241_10211972497884703_5192859875848

$\boxed{\text{Bài 2a}}$

Giả sử tồn tại pt $x^2+ax+b=0$ thỏa mãn điều kiện đề bài. Gọi 2 nghiệm của pt này là $x_1,x_2$.

Khi đó ta có:$\left\{\begin{matrix} x_1x_2 & = & b\\ x_1+x_2 & = & -a \end{matrix}\right.$

Khi tăng hệ số của pt trên lên 1 đơn vị, ta dc pt mới:

$2x^2+(a+1)x+b+1=0$

Khi đó pt có 2 nghiệm là $x_1+1$ và $x_2+1$.

Theo Vi-et, ta có:$\left\{\begin{matrix} (x_1+1)+(x_2+1) & = & -\frac{a+1}{2}\\ (x_1+1)(x_2+1) & = & \frac{b+1}{2} \end{matrix}\right.$

Ta có $-\frac{a+1}{2}=x_1+x_2+2=2-a \Rightarrow a=5$

$\frac{b+1}{2}=x_1x_2+x_1+x_2+1=b-a+1=b-4 \Rightarrow b=9$.

Khi đó pt cần tìm là $x^2+5x+9=0$. Pt này vô nghiệm. Vậy ko tồn tại pt thỏa mãn yêu cầu đề bài.




#675795 $P=9x^2+8y^2-12xy+6x-20y+18$

Đã gửi bởi Kagome on 31-03-2017 - 12:23 trong Bất đẳng thức và cực trị

Ta có: $P=9x^2+8y^2-12xy+6x-20y+18=(3x-2y+2)^2+(2y-3)^2+5\geq 5$

Dấu "=" xảy ra khi: $\left\{\begin{matrix} x=\frac{1}{3}\\ y=\frac{3}{2} \end{matrix}\right.$

Mình có bấm máy thử thì thấy khi x=1,y=2 thì p=1.




#675779 $P=9x^2+8y^2-12xy+6x-20y+18$

Đã gửi bởi Kagome on 30-03-2017 - 23:55 trong Bất đẳng thức và cực trị

Cho hai số thực $x,y$. Tìm giá trị nhỏ nhất của biểu thức:

$P=9x^2+8y^2-12xy+6x-20y+18$




#675168 Đề thi HSG Toán lớp 9 huyện Châu Thành tỉnh Long An

Đã gửi bởi Kagome on 23-03-2017 - 20:28 trong Tài liệu - Đề thi

Bài 3: (5 điểm)

     Cho tam giác ABC có ba góc nhọn nội tiếp trong đường tròn (O; R), hai đường cao BE và CF của tam giác cắt nhau tại H. Kẻ đường kính AK của đường tròn (O; R), gọi I là trung điểm của BC.

     1) Chứng minh bốn điểm B, F, E, C cùng thuộc một đường tròn. Xác định tâm của đường tròn ngoại tiếp tứ giác BFEC.

     2) Chứng minh rằng: AH = 2.IO.

     3) Biết $\widehat{BAC}=60^{o}$, tính độ dài dây BC theo R.

Capture.PNG

2) $\angle BKC+\angle BAC=180^{\circ}$

    $\angle BHC+\angle BAC=180^{\circ}$

$\Rightarrow \angle BKC=\angle BHC$                           (1)

$\angle ABK=\angle ACK=90^{\circ}$ (góc nội tiếp chắn nửa đường tròn)

$\angle ABE=\angle ACF$ (cùng phụ $\angle A$)

$\Rightarrow \angle HBK=\angle HCK$                          (2)

(1)(2)$\Rightarrow BHCK$ là hình bình hành

$\Rightarrow H,I,K$ thẳng hàng và $IH=IK$

Mà $OA=OK\Rightarrow AH=2OI$.

3)$BC=R\sqrt{3}$.




#674740 Đề học sinh giỏi tỉnh Đồng Tháp 2016-2017

Đã gửi bởi Kagome on 19-03-2017 - 16:04 trong Tài liệu - Đề thi

Bài 5: Cho tam giác ABC nhọn nội tiếp đường tròn tâm O. Gọi E, F lần lượt là chân đường cao kẻ từ C và B của tam giác ABC. D là điểm đối xứng của A qua O, M là trung điểm BC, H là trực tâm tam giác ABC

            a) Chứng minh rằng M là trung điểm HD

            b) Gọi L là giao điểm thứ hai của CE với đường tròn tâm O. Chứng minh rằng H, L đối xứng nhau qua AB

Capture.PNG

a)$D$ đối xứng $A$ qua $O\Rightarrow D\in O\Rightarrow \angle BAC+\angle BDC=180^{\circ}$

$\angle BHC=\angle EHF$ 

$\angle EHF+\angle BAC=180^{\circ}$

$\Rightarrow \angle BDC=\angle BHC$      (1)

Mặt khác: $AOD$ là đường kính đường tròn $(O)$.

$\Rightarrow \angle ABD=\angle ACD=90^{\circ}$

Mà $\angle ABF=\angle ACE$

$\Rightarrow \angle HBD=\angle HCD$      (2)

$(1)(2)\Rightarrow HBDC$ là hình bình hành

$\Rightarrow đpcm$

b)$\angle EHA=\angle EFA=\angle ABC=\frac{1}{2}\text{sđ}\overarc{AC}=\angle ALC$

$\Rightarrow \triangle ALH$ cân tại $H\Rightarrow đpcm$.




#674600 Mở rộng bài T3 Tạp chí Toán học và tuổi trẻ số 477 tháng 1 2017

Đã gửi bởi Kagome on 17-03-2017 - 23:10 trong Bất đẳng thức và cực trị

Lời giải bài 4: Lê Khánh Sỹ

17354956_1234202590034413_177113652_n.jp

Cái chỗ AM suy rộng là sao vậy bạn? Bạn giải thích rõ hơn dược ko?